How to Solve Thin Film Interference Problems in Physics

แชร์
ฝัง
  • เผยแพร่เมื่อ 1 พ.ค. 2023
  • An explanation of thin film interference, constructive interference, destructive interference, and index of refraction, and then some practice problems.
    More videos at
    www.danthetutor.com/
    Patreon Link (recurring donations):
    www.patreon.com/user?u=84517584
    Paypal Link (one-time donations):
    www.paypal.com/paypalme/danth...

ความคิดเห็น • 63

  • @Matt-Holdren
    @Matt-Holdren 10 หลายเดือนก่อน +29

    Ive been stuck for literally hours, your video cleared it up in 30 min. Thank you!

    • @danthetutor2624
      @danthetutor2624  10 หลายเดือนก่อน +3

      Happy to help! Thanks for watching!

  • @sahejdhaliwal1039
    @sahejdhaliwal1039 4 หลายเดือนก่อน +8

    I have watched like a dozen videos on thin film interference and this is by far the best one. Builds such a good foundation on the subject, i somehow got all my homework after this

    • @danthetutor2624
      @danthetutor2624  4 หลายเดือนก่อน +1

      Wow, thanks for the compliment! Where's the award for "Best video on Thin Film Interference"?

  • @jamespereira4419
    @jamespereira4419 หลายเดือนก่อน +2

    Watching this 2 hours before the AP Phys 2 exam, this helped so much!

  • @massimom68
    @massimom68 9 หลายเดือนก่อน +4

    😀I join Matt-Holdren's comment. I've been stuck trying to solve one exercise for hours, but thanks to your video I was finally able to solve it. The formula and solution method you showed are very clear and easy to remember. I am wondering why it's not explained so clearly in my physics book... Thank you very much!!!

  • @radeshl7441
    @radeshl7441 หลายเดือนก่อน +1

    Thankyou so much sir!!. Best video for film interference

  • @sunaymukherjee7576
    @sunaymukherjee7576 3 หลายเดือนก่อน +1

    Thank god I found this in time man...my head was spinning as to why we were switching the formula of constructive and destructive this whole time... Thanks a lot!

    • @danthetutor2624
      @danthetutor2624  3 หลายเดือนก่อน

      Believe me, I wish I knew this sooner too, haha

  • @aechwithana2349
    @aechwithana2349 7 วันที่ผ่านมา

    Thank you so much Dan. You've saved my life.

    • @danthetutor2624
      @danthetutor2624  7 วันที่ผ่านมา

      No problem! Next time, you can save my life.

  • @emreakgul4394
    @emreakgul4394 6 หลายเดือนก่อน +2

    What a brilliant and straight forward explanation, you're saving lives !

    • @danthetutor2624
      @danthetutor2624  6 หลายเดือนก่อน

      I’m not a doctor. I’m just your normal everyday supertutor.

  • @ximenazuniga8436
    @ximenazuniga8436 7 หลายเดือนก่อน

    this was so helpful, thank you so much! I have my physics midterm in a couple hours

  • @yasserahmed2812
    @yasserahmed2812 ปีที่แล้ว +1

    Simple and beautiful explanation!

  • @emmadevries4731
    @emmadevries4731 หลายเดือนก่อน

    best video out there on thin film interference. period. thank you a trillion

    • @danthetutor2624
      @danthetutor2624  หลายเดือนก่อน

      Thanks for the stellar review!

  • @Xx-ee1ui
    @Xx-ee1ui ปีที่แล้ว

    i finally understand now !!! thank you!

  • @matthewcheah3178
    @matthewcheah3178 หลายเดือนก่อน

    Thank you!

  • @wallyalshamari3526
    @wallyalshamari3526 10 หลายเดือนก่อน

    great video thanks for the help!

  • @everette4111
    @everette4111 2 หลายเดือนก่อน

    absolute guy for this

  • @marsyaalea123
    @marsyaalea123 7 หลายเดือนก่อน

    thanksss i understood it very well !!

  • @perihangummadova5017
    @perihangummadova5017 5 หลายเดือนก่อน

    Best tutorr everrr.. i dua you all the best hocamm

  • @lefishe8145
    @lefishe8145 ปีที่แล้ว +1

    Great explanation, thank you I finally understand this now lol

  • @tavongamukhachana9053
    @tavongamukhachana9053 ปีที่แล้ว

    the best love this

  • @MetagesAschalew
    @MetagesAschalew ปีที่แล้ว

    Best teaching

  • @kaihungwong
    @kaihungwong 25 วันที่ผ่านมา

    thanks sir, good vid

  • @filippofonseca
    @filippofonseca 4 หลายเดือนก่อน

    legend!

  • @vincentxayasak8773
    @vincentxayasak8773 2 หลายเดือนก่อน

    Thank u, u saved my life before finals

    • @danthetutor2624
      @danthetutor2624  2 หลายเดือนก่อน

      No prob. Now go crush that exam!

  • @EricJeon0529
    @EricJeon0529 หลายเดือนก่อน

    perfect teacher and better then our school teacher

    • @danthetutor2624
      @danthetutor2624  หลายเดือนก่อน

      Really setting a low bar there… haha thanks!

  • @shubkarmanbhullar6784
    @shubkarmanbhullar6784 ปีที่แล้ว

    brilliant

  • @t.adalaa
    @t.adalaa 5 หลายเดือนก่อน

    Dan the tutor❤️

  • @sergiodevicentesanluis9741
    @sergiodevicentesanluis9741 5 หลายเดือนก่อน

    Why is it that when you go from high to low n there isn't a phase shift? 3:35

  • @quangmindang1130
    @quangmindang1130 19 วันที่ผ่านมา

    niceeee

  • @leo-yt8jg
    @leo-yt8jg 8 หลายเดือนก่อน

    Ur the best

  • @andrewjustin256
    @andrewjustin256 9 หลายเดือนก่อน

    2:25 Could you please inform me why we need the smallest value possible and when will we use the greater values?

    • @danthetutor2624
      @danthetutor2624  9 หลายเดือนก่อน +1

      Good question. Usually, the question has some wording like "minimum thickness", and that means that you use the smallest value for m. You'll notice that my questions did not say that, so technically you can use the larger values for this video.
      The only time you NEED the larger values is when the question says something like "what are the first 3 thicknesses that will work?" In that case, you'd use the first 3 values for m (for instance m = 1, 2, 3)

    • @andrewjustin256
      @andrewjustin256 9 หลายเดือนก่อน

      @@danthetutor2624 But why do we need the greater values in the second scenario; I don't have an intuitive feeling for it. Besides, I don't walk with you in the notion of "you can technically use it". What do you exactly mean by this? Will I get the same results even if I use larger values?

    • @danthetutor2624
      @danthetutor2624  9 หลายเดือนก่อน

      It's because there are infinitely many answers to the thickness question, depending on what value you use for m. So for instance the answer could be 1 meter thick, 2 meters thick, 3 meters thick, etc. That's why most problems will say "minimum thickness".
      I say "technically" because I meant to put in the word "minimum thickness" but I forgot. So any value of m would work. But most problems will not forget to say that.

  • @chandlerwoo4091
    @chandlerwoo4091 7 หลายเดือนก่อน

    Wouldn’t you have to change your wavelength to the wavelength in the film?

    • @danthetutor2624
      @danthetutor2624  7 หลายเดือนก่อน

      I see what you're saying. I think that makes logical sense, but every physics problem I've ever seen doesn't do that. So I'm inclined to say "no, don't change the wavelength" but I don't have a good reason why.

  • @user-xo5co5ew1j
    @user-xo5co5ew1j 8 หลายเดือนก่อน

    There's a point I don't really get it. How can I know there is actually 1 lambda in 2t? if light is blocked out, there's probably 2 lambda in 2t, therefore the answer of thickness might be tow times of yours

    • @danthetutor2624
      @danthetutor2624  8 หลายเดือนก่อน

      Which problem are you talking about?

    • @user-xo5co5ew1j
      @user-xo5co5ew1j 7 หลายเดือนก่อน

      @@danthetutor2624 thin film interference thickness measurement

  • @hussienbast2342
    @hussienbast2342 2 หลายเดือนก่อน +1

    Most other videos the wavelength in the formula is taken as the wavelength of the film ,, why you take it as the wavelength of the light???

    • @ryanjiang1376
      @ryanjiang1376 หลายเดือนก่อน

      I was bouta ask the same question

    • @ryanjiang1376
      @ryanjiang1376 หลายเดือนก่อน

      @ Dan the tutor

    • @ryanjiang1376
      @ryanjiang1376 หลายเดือนก่อน

      @hussienbast2342 collegeboard says to use wavelength of film so I’d just do thT

  • @robertdarling2071
    @robertdarling2071 หลายเดือนก่อน

    For us non physics people, what exactly is a phase shift?

    • @danthetutor2624
      @danthetutor2624  หลายเดือนก่อน +1

      Let me bestow some knowledge to you: light moves like a wave. Just like a sin or cos wave from math class. And so when we say there’s a phase shift, we’re talking about moving the sin graph by exactly half a period (half a cycle). It doesn’t matter if you move the graph left or right because it’s symmetric and it will look the same no matter what.

    • @robertdarling2071
      @robertdarling2071 หลายเดือนก่อน

      Thank you much!

  • @user-fp7qt5gj6g
    @user-fp7qt5gj6g 2 หลายเดือนก่อน

    save my life

  • @shmsaaa
    @shmsaaa ปีที่แล้ว

    mwa 💋

  • @realdbcooper3423
    @realdbcooper3423 5 หลายเดือนก่อน

    He didnt explain anything

    • @aidencharles3106
      @aidencharles3106 3 หลายเดือนก่อน

      He explained everything lol